You are on page 1of 54

PhysicsWOOT (2751)

AoPS Staff

Tuesday
Aug 24, 2021 - Apr 12, 2022
7:30 - 9:30 PM ET (4:30 - 6:30 PM PT)

Overview
Lesson 2 (Sep 14) Class Transcript - Newton's Laws
< Go back to the class overview page
Copyright © AoPS Incorporated. This page is copyrighted material. You can view and print this page for your own use, but you
cannot share the contents of this file with others.
Display all student messages • Show few student messages • Hide student messages
meichenlaub 2021-09-14 19:30:23
Newton's Laws of Motion

meichenlaub 2021-09-14 19:30:26


This week we'll study the core of mechanics - Newton's laws. You should already be quite familiar with Newton's three laws, so we'll
spend our time on problem-solving.

meichenlaub 2021-09-14 19:30:34


Two people stand on opposite ends of a long sled on frictionless ice. The sled is oriented in the east-west direction, and
everything is initially at rest. The western person then throws a ball eastward toward the eastern person, who catches it. The sled

meichenlaub 2021-09-14 19:30:46


a) moves eastward, and then ends up at rest

b) moves eastward, and then ends up moving westward

c) moves westward, and then ends up at rest

d) moves westward, and then ends up moving eastward

e) does not move at all

meichenlaub 2021-09-14 19:30:53


Let's break this question down just a little bit. What direction does the sled move after the second person catches the ball?

meichenlaub 2021-09-14 19:31:20


(And also explain how you know.)

mathisawesome01 2021-09-14 19:32:08


stops because the net motion must be 0

Zian2006 2021-09-14 19:32:08


It stops. Conservation of momentum

mathtiger6 2021-09-14 19:32:08


will be at rest because the momentum changes from the throw and the catch cancel.

mathisawesome01 2021-09-14 19:32:08


stops because v CM must be 0

Pleaseletmewin 2021-09-14 19:32:08


it stops, by conservation of momentum

EthanTLee23 2021-09-14 19:32:08


The sled does not move at all, because the ball if it was moving, it would violate conservation of momentum
s3v3n 2021-09-14 19:32:08
A bit to the east, since momentum has been transferred in the eastward direction

meichenlaub 2021-09-14 19:32:14


The sled doesn't move at all after the second person catches the ball.

meichenlaub 2021-09-14 19:32:24


As many of you pointed out, we can use conservation of momentum to explain that.

meichenlaub 2021-09-14 19:32:33


We can say essentially the same thing with Newton's first law.

meichenlaub 2021-09-14 19:32:40


\This law says that the center of mass of a system doesn't accelerate unless there are external forces.

meichenlaub 2021-09-14 19:32:45


Initially, the velocity of the center of mass of the sled system is zero. There are no external forces, so the velocity of the center of
mass stays zero. That means that when the second person catches the ball, everything is stationary.

meichenlaub 2021-09-14 19:33:01


Another way to see it is with Newton's third law. When a person exerts a force on the ball, the ball exerts a force on the person in
the opposite direction. Suppose the sled wound up drifting to the right. Then the ball picked up some momentum to the right.
Because the change in momentum of the ball is

Δp ball = ∫ F net; balldt,

we know that the net force on the ball, integrated over time, was to the right.

meichenlaub 2021-09-14 19:33:18


The same reasoning applies to the people/sled. They pick up momentum to the right, so the net force on them, integrated over
time, was to the right.

meichenlaub 2021-09-14 19:33:24


But at any given time, the a force to the right on the ball implies an equal-magnitude force to the left on the people / sled. So
there's no way for both the ball and the sled to pick up momentum to the right (or to the left). So they must stay put over time.

meichenlaub 2021-09-14 19:33:39


So we can explain a situation using various combinations of Newton's laws. Some lines of reasoning may be more direct, but it's
good to be able to find different ones.

meichenlaub 2021-09-14 19:33:50


Okay, we've ruled out (b) and (d), which say the carts ends up with some motion. Which answer is correct and why?

mathisawesome01 2021-09-14 19:34:42


c

EthanTLee23 2021-09-14 19:34:42


c

Zian2006 2021-09-14 19:34:42


C. Again conservation of momentum

mathtiger6 2021-09-14 19:34:42


C because throwing eastward causes the sled to go the other way.

EL2024 2021-09-14 19:34:42


c , iniitally also because of conservation of momentum since the ball moves east the sled must move west

PhysKid11 2021-09-14 19:34:42


c) also because of conservation of momentum

Hiboy 2021-09-14 19:34:42


c is correct because equal/opposite force by the wester person
Dodgers66 2021-09-14 19:34:42
C since the original throw gives ball eastward momentum

DGA_Student_1 2021-09-14 19:34:42


(c), because when the ball is moving eastward, the sled must be moving westward to make the total linear momentum zero

Latnor 2021-09-14 19:34:42


C, since the sled moves in the opposite direction of the ball (closed system means net momentum of all objects remains the same)

meichenlaub 2021-09-14 19:35:01


It's (c): the sled moves westward, and then ends up at rest. We said earlier that the center of mass of the system begins with
velocity zero and that it doesn't accelerate. This means that the center of mass doesn't move at all. The ball is moving from west to
east, so the cart must move from east to west while the ball is in the air.

meichenlaub 2021-09-14 19:35:12


You could also use momentum conservation - the sled starts with zero momentum so it ends with zero momentum. This is
basically the same thing as Newton's first law. Because the momentum of a system is its mass times the velocity of its center of
mass, saying that the momentum stays zero during a process and saying that the center of mass doesn't move during a process
are equivalent.

meichenlaub 2021-09-14 19:35:17


Here's a classic problem:

meichenlaub 2021-09-14 19:35:19


A cart of mass M is filled with water with total mass m and sits on a frictionless rail. A hole is drilled in the bottom of the left-hand
side of the cart. There is a pump connected to the hole which is initially off, so that the cart and water in it are all stationary. At
time t = 0, the pump operates so that mass of water draining through the hole per unit time is f. The internal dynamics of the cart
ensure that the water surface remains flat and horizontal at all times. The water drains so that, as viewed from the cart, it moves
directly downward while it exits the cart. Describe the motion of the cart over time.

meichenlaub 2021-09-14 19:35:21

meichenlaub 2021-09-14 19:36:22


(Here I've introduced some simplifying assumptions; other versions of the problem don't make the assumption that the flow rate is
constant or that the water surface stays flat.)

meichenlaub 2021-09-14 19:36:25


Let's begin with a simple question: does the cart move at all? Please explain how you know.

EthanTLee23 2021-09-14 19:38:10


Yes, because when the water falls, it falls on the left side. This shifts the center of mass to the left

s3v3n 2021-09-14 19:38:10


The cart should move, because the distribution of the mass of water is changing

taniaban 2021-09-14 19:38:10


yes because otherwise the center of mass would change

Zian2006 2021-09-14 19:38:10


Yes. The center of mass shifts since the hole is drilled off-center

meichenlaub 2021-09-14 19:38:19


Yes, the cart does move. Just like in the previous problem, there are no external horizontal forces on the cart+water system in this
problem. That means the center of mass of the cart+water system can't accelerate in the horizontal direction, and because the
center of mass begins with zero horizontal velocity, we conclude that the center of mass does not move at all in the horizontal
direction.

meichenlaub 2021-09-14 19:38:36


Suppose the cart remained stationary as the water drained. Then the center of mass of the water would move from the center of
the cart towards the left, where the hole is, as the water drained. Since the cart remains stationary, the center of mass of the
water+cart system moves to the left in this scenario. But that's impossible!

meichenlaub 2021-09-14 19:39:19


(The center of mass also moves downward, but we aren't concerned with the vertical direction and can ignore it.)

meichenlaub 2021-09-14 19:39:27


So which way does the cart have to move as the water begins to drain?

DGA_Student_1 2021-09-14 19:40:12


to the right

Pleaseletmewin 2021-09-14 19:40:12


to the right

taniaban 2021-09-14 19:40:12


right

Hershey806 2021-09-14 19:40:12


right

Zian2006 2021-09-14 19:40:12


Right

harinsrikanth 2021-09-14 19:40:12


To the right

mathtiger6 2021-09-14 19:40:12


to the right

Hiboy 2021-09-14 19:40:12


to the right

s3v3n 2021-09-14 19:40:12


To the right

EL2024 2021-09-14 19:40:12


right

EthanTLee23 2021-09-14 19:40:12


The cart moves to the right

aj1745 2021-09-14 19:40:12


horizontally to the right

mathisawesome01 2021-09-14 19:40:12


right

meichenlaub 2021-09-14 19:40:27


The cart has to move to the right. This is because the water is moving to the left to get to the hole. (It's just like the ball and the
sled problem we did before.)

meichenlaub 2021-09-14 19:40:39


If the cart moves to the right after beginning stationary, it must have some acceleration to the right, which means there must be
some force on it to the right. What exerts a rightward force on the cart?

EL2024 2021-09-14 19:41:30


the water?
EthanTLee23 2021-09-14 19:41:30
A force coming from the water?

s3v3n 2021-09-14 19:41:30


The water within the cart?

mathtiger6 2021-09-14 19:41:30


water pressure

DGA_Student_1 2021-09-14 19:41:30


the water

mathisawesome01 2021-09-14 19:41:30


the water exiting

taniaban 2021-09-14 19:41:30


the water

meichenlaub 2021-09-14 19:41:52


Only the water can do that! The rail is frictionless. The cart exerts a leftward force on the water, accelerating it towards the hole,
and by Newton's third law, the water exerts a rightward force on the cart.

meichenlaub 2021-09-14 19:41:56


In fact, if you work out the mathematical theory later, you'll see that the force delivered to the cart is impulsive, meaning that the
momentum is given to the cart all at once. In order to accelerate the water from zero velocity to a finite velocity to the left in just an
instant, the force from the walls of the cart on the water must be infinitely large, so the cart starts moving rightward in a sudden
jolt.

meichenlaub 2021-09-14 19:42:19


You can take it as a fact that the cart winds up moving rightward at constant speed while draining.

meichenlaub 2021-09-14 19:42:24


Next question: can the cart continuously move to the right forever? Explain how you get your answer in addition to saying what
your answer is.

EthanTLee23 2021-09-14 19:43:54


No. Eventually, all the water will drain out of the cart. If the cart were to keep going, then the center of mass would shift to the right,
which is not allowed

Zian2006 2021-09-14 19:43:54


No. Otherwise there will be a final net momentum to the right

EL2024 2021-09-14 19:43:54


no for the cart to move rightward forever the water would have to move leftward forever

s3v3n 2021-09-14 19:43:54


If it keeps moving, then all the water and the cart end up moving to the right after starting at rest which violates conservation of
momentum

Dodgers66 2021-09-14 19:43:54


no because all the water plus the car twould move right, violating conservation of momentum. Intuitively, something probably
happens when all the water exits just like when it starts exiting

mathtiger6 2021-09-14 19:43:54


no, the center of mass would be too far right

meichenlaub 2021-09-14 19:44:14


Nope, that is not possible. As the cart begins moving to the right, the water draining out also moves to the right outside of the cart
(because the problem states that the water leaving the cart has no horizontal velocity relative to the cart.) So if the cart kept
moving to the right forever, we'd have the cart moving to the right and the water moving to the right. The center of mass of the
system would be moving to the right, and we know it should be stationary.

meichenlaub 2021-09-14 19:44:35


Instead, just as the last bit of water is draining out, there is one more sudden impulse on the cart.
meichenlaub 2021-09-14 19:44:39
So all told, what's a qualitative description of the motion of the cart?

meichenlaub 2021-09-14 19:46:06


Most people are telling me the cart comes to a stop.

meichenlaub 2021-09-14 19:46:16


I think we are not entirely clear on what the water does.

meichenlaub 2021-09-14 19:46:34


When the water is *inside* the cart, on average it has to move left, relative to the cart, to get to the hole.

meichenlaub 2021-09-14 19:46:58


But that is a separate issue from what the water does at it leaves the cart. The problem states that the water falls straight down
from the point of view of the cart.

meichenlaub 2021-09-14 19:47:13


So if the cart is moving to the right as it drains, the water leaving the cart also moves to the right.

Zian2006 2021-09-14 19:47:44


Right, then left

bananaman321 2021-09-14 19:47:44


It goes the other way

DGA_Student_1 2021-09-14 19:47:44


it is moving left at the end, as the water is still moving right

EL2024 2021-09-14 19:47:44


the waters moving right so the cart goes left?

EL2024 2021-09-14 19:47:44


so the cart moves right, then left when the water drains

meichenlaub 2021-09-14 19:47:53


A lot of people are still saying the cart stops once the water is gone. But that can't be. Then you'd have the water that drained out of
the cart moving to the right and the cart stationary. The center of mass of the system would be moving to the right.

meichenlaub 2021-09-14 19:47:56


When the water begins draining, the cart first moves to the right. Then, as the last bit of water drains out, the cart switches
directions and goes to the left. So in the end, the water moves to the right forever and the cart drifts to the left forever, and the
center of mass never moves left or right at all.

meichenlaub 2021-09-14 19:48:05


What exerts a force on the cart to change its direction?

Zian2006 2021-09-14 19:49:14


The water again

s3v3n 2021-09-14 19:49:14


The water again?

aj1745 2021-09-14 19:49:14


the force of the water leaving?

DGA_Student_1 2021-09-14 19:49:14


the water

HappiHippo 2021-09-14 19:49:14


is it the water pressure (or lack thereof?)

Chickenmom 2021-09-14 19:49:14


last bit of water draining

meichenlaub 2021-09-14 19:49:18


It's the water. Specifically, it's the water that's just getting to the hole. We said earlier that the water inside the cart moves to the left
to get to the hole. Then when the water gets to the hole, it suddenly changes directions, moving to the right to keep up with the
cart. So the water must have experienced a force to the right from the cart just as it is exiting the cart. By Newton's third law, the
water exerts a large force to the left on the cart. This is the force that winds up changing the direction of the cart's motion.

meichenlaub 2021-09-14 19:49:38


We aren't going to analyze this problem in full mathematical detail here, but feel free to give it a shot after class and post your
results on the message board!

meichenlaub 2021-09-14 19:50:17


Next we'll look at a problem that was on USAPhO in 2012.

meichenlaub 2021-09-14 19:50:22


A particle of mass m moves under a force similar to that of an ideal spring, except that the force repels the particle from the origin:

F = + mα 2x.

The position of the particle as a function of time can be written

x(t) = Af 1(t) + Bf 2(t)

for some appropriate functions f 1 and f 2.

meichenlaub 2021-09-14 19:50:29


f 1(t) and f 2(t) can be chosen to have the form e rt. What are the two appropriate values of r ?

meichenlaub 2021-09-14 19:50:40


What law are we going to use here? Also state what that law is as an equation.

mathtiger6 2021-09-14 19:51:48


F = ma

Zian2006 2021-09-14 19:51:48


Newton's 2nd: F=ma

AOPS81619 2021-09-14 19:51:48


F = mẍ

HappiHippo 2021-09-14 19:51:48


second law? F=ma

Dodgers66 2021-09-14 19:51:48


newton's second law: F = mẍ

s3v3n 2021-09-14 19:51:48


Newton's Second, F = ma

DGA_Student_1 2021-09-14 19:51:48


F = mẍ

EL2024 2021-09-14 19:51:48


newtons second law, F=ma

Pleaseletmewin 2021-09-14 19:51:48


d 2x
F=m ; we can also guess that x is an exponential function as well
dt 2

aj1745 2021-09-14 19:51:48


The 2nd law. F=ma

Nivek 2021-09-14 19:51:48


F=mx double dot
QMech 2021-09-14 19:51:48
Newton's 2nd law, F=mẍ

Dhillonr25 2021-09-14 19:51:48


F=ma?

PhysKid11 2021-09-14 19:51:48


f=ma, second law

meichenlaub 2021-09-14 19:51:53


This is a straight-up Newton's second law question. We are given a force, and Newton's second law tells us how particles will move
under a given force.

meichenlaub 2021-09-14 19:51:58


→ →
In mathematical problem-solving, the second law, F net = m a , often does the heavy lifting. The second law gives us an equation of
motion, an equation (often a differential equation) for the position of an object as a function of time. This problem illustrates that
basic idea.

meichenlaub 2021-09-14 19:52:13


Because the motion is one-dimensional, the vector nature of the forces and accelerations is captured with a simple positive or
negative sign. (We focus much more on non-trivial multidimensional problems with vectors next year in PhysicsWOOT.)

meichenlaub 2021-09-14 19:52:30


If we plug the force given in the problem into Newton's second law F = ma, we have

mα 2x = ma,

or canceling the mass

α 2x = a.

meichenlaub 2021-09-14 19:52:53


a, the acceleration, is the second derivative of the position with respect to time,

d 2x
a= .
dt 2

meichenlaub 2021-09-14 19:52:57


Plugging that into the equation α 2x = a, we get

meichenlaub 2021-09-14 19:52:58

d 2x
α 2x = .
dt 2

meichenlaub 2021-09-14 19:53:03


This is the equation of motion for this system.

meichenlaub 2021-09-14 19:53:12


Sometimes, we solve an equation of motion directly, but another technique is to guess what the answer is. What guess should we
use?

Zian2006 2021-09-14 19:54:13


rt
e

s3v3n 2021-09-14 19:54:13


An exponential function

Hiboy 2021-09-14 19:54:13


e^(alpha*t)
taniaban 2021-09-14 19:54:13
exponential

AOPS81619 2021-09-14 19:54:13


Use the x(t) = Ae rt that we are given

HappiHippo 2021-09-14 19:54:13


e αt

EL2024 2021-09-14 19:54:13


e^alphat

PhysKid11 2021-09-14 19:54:13


Ae^rt + Be^rt

DGA_Student_1 2021-09-14 19:54:13


an exponential solution of the form Ae rt

HappiHippo 2021-09-14 19:54:13


and the negative version of that too

mathisawesome01 2021-09-14 19:54:13


x = Ae rt + Be − rt

Zian2006 2021-09-14 19:54:13


r 1t r 2t
Or Ae + Be

meichenlaub 2021-09-14 19:54:20


As a guess for the solution to the equation of motion, we should use the equation given in the problem,

x = Ae r At + Be r Bt.

meichenlaub 2021-09-14 19:54:28


d 2x
So what's in terms of A, B, r A, r B, e and t ?
dt 2

meichenlaub 2021-09-14 19:54:35


If it's too much to type, you can just say "done" when you have it.

Pleaseletmewin 2021-09-14 19:55:57

Ar 2Ae r At + Br 2Be r Bt

Dodgers66 2021-09-14 19:55:57


r 2AAe r At + r 2BBe r Bt

mathisawesome01 2021-09-14 19:55:57


Ar 2Ar r At + Br 2Be r Bt

mathtiger6 2021-09-14 19:55:57


Ar 2Ae r At + Br 2Be r Bt

PhysKid11 2021-09-14 19:55:57


Ar_A^2 e^(r_At) + Br_B^2 e^(r_Bt)

DGA_Student_1 2021-09-14 19:55:57


Ar 2Ae r At + Br 2Be r Bt

AOPS81619 2021-09-14 19:55:57


Ar 2Ae r At + 2 r Bt
Br Be

mathisawesome01 2021-09-14 19:55:57


2 r At 2 r Bt
Ar A e + Br B e
Latnor 2021-09-14 19:55:57
d 2x
α 2(Ae r At + Be r Bt) =
dt 2

bfan0805 2021-09-14 19:55:57


2 2
Ar Ae r At + Br Be r Bt

EthanTLee23 2021-09-14 19:55:57


2 2
Ar Ae r At + Br Be r Bt

meichenlaub 2021-09-14 19:56:03


Taking two derivatives,

d 2x 2 2
= r AAe r At + r BBe r Bt.
dt 2

meichenlaub 2021-09-14 19:56:10


d 2x
Plugging our equation for and our equation for x into the equation of motion, we get
dt 2

meichenlaub 2021-09-14 19:56:14

( )
α 2 Ae r At + Be r Bt = r 2AAe r At + r 2BBe r Bt.

meichenlaub 2021-09-14 19:56:23


And can you find the values of r A and r B ?

mathtiger6 2021-09-14 19:57:24


±α

EL2024 2021-09-14 19:57:24


plus or minus alpha

HappiHippo 2021-09-14 19:57:24


⟹ r A, r B = ± α

Zian2006 2021-09-14 19:57:24


±α

bfan0805 2021-09-14 19:57:24


±α

s3v3n 2021-09-14 19:57:24


α and − α

Dodgers66 2021-09-14 19:57:24


±α

AOPS81619 2021-09-14 19:57:24


r 2A 2
= α and r 2B = α 2, so r A and r B are ± α

Hiboy 2021-09-14 19:57:24


+-alpha

meichenlaub 2021-09-14 19:57:33


The above equation should work for any values of A and B. Specifically, we could let B = 0. Then the equation simplifies to

α 2Ae r At = r 2AAr r At.

meichenlaub 2021-09-14 19:57:38


That means

2
α2 = r A

and

r A = ± α.

meichenlaub 2021-09-14 19:57:43


Exactly the same reasoning works for r B with the same result, so we can choose

meichenlaub 2021-09-14 19:57:44

r A = α; r B = − α.

meichenlaub 2021-09-14 19:57:50


(You could choose to switch them. But if you choose + for both, you're throwing out an important solution.)

meichenlaub 2021-09-14 19:58:00


That's the first part.

meichenlaub 2021-09-14 19:58:02


Suppose that the particle begins at position x(0) = x 0 and with velocity v(0) = 0. What is x(t) ?

meichenlaub 2021-09-14 19:58:11


We can write x as

meichenlaub 2021-09-14 19:58:13

x = Ae αt + Be − αt.

meichenlaub 2021-09-14 19:58:16


We are given two facts, x(0) = x 0 and v(0) = 0. What equation does the first fact give you about A and B ?

PhysKid11 2021-09-14 19:59:28


A+B=x_0

mathtiger6 2021-09-14 19:59:28


A + B = x0

s3v3n 2021-09-14 19:59:28


A + B = x0

HappiHippo 2021-09-14 19:59:28


A + B = x0

bfan0805 2021-09-14 19:59:28


A + B = x0

Hiboy 2021-09-14 19:59:28


A+B = x_0

theallpro1 2021-09-14 19:59:28


A+B=x_0

mathisawesome01 2021-09-14 19:59:28


A + B = x0

DGA_Student_1 2021-09-14 19:59:28


A + B = x0

AOPS81619 2021-09-14 19:59:28


A + B = xo

EL2024 2021-09-14 19:59:28


A+B=x0

meichenlaub 2021-09-14 19:59:32


Plugging in t = 0,

meichenlaub 2021-09-14 19:59:34

x0 = A + B

meichenlaub 2021-09-14 19:59:39


And what equation does the second fact give you for A and B ?

PhysKid11 2021-09-14 20:00:29


A−B=0

bfan0805 2021-09-14 20:00:29


A=B

mathtiger6 2021-09-14 20:00:29


Aα − Bα = 0 ⟹ A = B

EL2024 2021-09-14 20:00:29


A=B

HappiHippo 2021-09-14 20:00:29


A−B=0

taniaban 2021-09-14 20:00:29


A−B=0

s3v3n 2021-09-14 20:00:29


A=B

DGA_Student_1 2021-09-14 20:00:29


A=B

Zian2006 2021-09-14 20:00:29


A=B

No_u_ 2021-09-14 20:00:29


A-B=0

Hiboy 2021-09-14 20:00:29


A=B

Hershey806 2021-09-14 20:00:29


A=B

meichenlaub 2021-09-14 20:00:36


Differentiating once, we get

v = αAe αt − αBe αt.

meichenlaub 2021-09-14 20:00:42


Plugging in t = 0 and v = 0 we get

meichenlaub 2021-09-14 20:00:44

0 = A − B.

meichenlaub 2021-09-14 20:00:51


So what are A and B ?

Zian2006 2021-09-14 20:01:40


A = B = x0 / 2

bfan0805 2021-09-14 20:01:40


x0
2

PhysKid11 2021-09-14 20:01:40


x_0/2

mathtiger6 2021-09-14 20:01:40


x0
both
2

mathisawesome01 2021-09-14 20:01:40


x0
A=B=
2

EL2024 2021-09-14 20:01:40


A=B=0.5x0

DGA_Student_1 2021-09-14 20:01:40


x0
A= 2 =B

s3v3n 2021-09-14 20:01:40


x0
A=B= 2

Hershey806 2021-09-14 20:01:40


x0
2

EthanTLee23 2021-09-14 20:01:40


x0
A=B= 2

HappiHippo 2021-09-14 20:01:40


x0
2

Dhillonr25 2021-09-14 20:01:40


x0/2

QMech 2021-09-14 20:01:40


(x_0)/2

AOPS81619 2021-09-14 20:01:40


x0
A=B= 2

meichenlaub 2021-09-14 20:01:43


Adding our two equations for A and B gives

x 0 = 2A.

and so

x0
A= .
2

meichenlaub 2021-09-14 20:01:47


Then solving either equation for B,
\

meichenlaub 2021-09-14 20:01:58

x0
B=
2

meichenlaub 2021-09-14 20:02:01


And finally what's x(t) ?

s3v3n 2021-09-14 20:03:33


x0
(e αt + e − αt)
2

bfan0805 2021-09-14 20:03:33


Ax 0 Bx 0
2
e αt + 2 e − αt

mathisawesome01 2021-09-14 20:03:33


x0
x(t) =
2 (
e αt + e − αt )
DGA_Student_1 2021-09-14 20:03:33
x0
(
x(t) = 2 e αt + e − αt )
Chickenmom 2021-09-14 20:03:33
x/2 e^alpha x t + x/2 e^- alpha time

QMech 2021-09-14 20:03:33


x(t) = (x_0)/2 * (e^-αt + e^αt)

Zian2006 2021-09-14 20:03:33


x0
(e αt + e − αt)
2

Hershey806 2021-09-14 20:03:33


x0
2
(e αt + e − αt)

EthanTLee23 2021-09-14 20:03:33


x0
x = 2 (e αt + e − αt)

Square_264 2021-09-14 20:03:33


x0
(e αt + e − αt)
2

meichenlaub 2021-09-14 20:03:41


It's

meichenlaub 2021-09-14 20:03:42

x0
x1 =
2 (e αt
)
+ e − αt .

meichenlaub 2021-09-14 20:04:05


(I've called it x 1 because we're about to look at another particle with a different initial condition, and we'll call that one's position x 2.
)

meichenlaub 2021-09-14 20:04:09


Another way to write this is

x 1 = x 0cosh(αt).
meichenlaub 2021-09-14 20:04:12
This defines the hyperbolic cosine function. The hyperbolic trig functions take the role of ordinary trig function near unstable
equilibria.

meichenlaub 2021-09-14 20:04:26


A second, identical particle begins at position x(0) = 0 with velocity v(0) = v 0. The second particle becomes closer and closer to
the first particle as time goes on. What is v 0?

meichenlaub 2021-09-14 20:04:47


Okay, can you find x(t) for this particle?

meichenlaub 2021-09-14 20:04:51


(This is kinda long. I'll give you a few minutes. Just try to do the same basic thing you just did with the new information.)

mathtiger6 2021-09-14 20:07:58


v0
(
x(t) = 2α e αt − e − αt )
Zian2006 2021-09-14 20:07:58
v0
x(t) = 2α (e − e − αt) αt

EthanTLee23 2021-09-14 20:07:58


v0
x 2 = 2α (e αt − e − αt)

Dodgers66 2021-09-14 20:07:58


v0
2α (e αt − e − αt )
PhysKid11 2021-09-14 20:07:58
sorry! meant x(t)=v_0/alpha * sinh(alpha t)

meichenlaub 2021-09-14 20:08:04


We run through all the same steps as before. The initial position x(0) = 0 tells us

A + B = 0.

meichenlaub 2021-09-14 20:08:08


The initial velocity v(0) = v 0 tells us

v0
A−B= .
α

meichenlaub 2021-09-14 20:08:16


Solving for A and B, we get

v0
A= .

v0
B= −

meichenlaub 2021-09-14 20:08:21


and for x we have

meichenlaub 2021-09-14 20:08:23

v0
x 2(t) =
2α (e αt
)
− e − αt .
meichenlaub 2021-09-14 20:08:29
v0
(This is a hyperbolic sine function, x 2 = sinh(αt). )
α

meichenlaub 2021-09-14 20:08:46


Next, we want to know about what happens after a long time. What can we do with x 1 and x 2 ?

taniaban 2021-09-14 20:09:41


set them equak

mathisawesome01 2021-09-14 20:09:41


subtract

taniaban 2021-09-14 20:09:41


set them equal

QMech 2021-09-14 20:09:41


take the limit as t approaches infinity

PhysKid11 2021-09-14 20:09:41


limit as t tends to infinty

mathisawesome01 2021-09-14 20:09:41


subtract x 2 from x 1

Zian2006 2021-09-14 20:09:41


Limit as t-> infinity equal

Hershey806 2021-09-14 20:09:41


set them equal

EthanTLee23 2021-09-14 20:09:41


Since they become closer to each other, we can set them to be equal?

DGA_Student_1 2021-09-14 20:09:41


− αt
the e term approaches zero at large t

Hiboy 2021-09-14 20:09:41


lim as t goes to inf

bfan0805 2021-09-14 20:09:41


they grow closer as t increases

EL2024 2021-09-14 20:09:41


e^-αt is small

mathisawesome01 2021-09-14 20:09:41


lim t → ∞(x 2 − x 1)

Dhillonr25 2021-09-14 20:09:41


find the limit as t approaches infinity

meichenlaub 2021-09-14 20:09:47


We want the positions to be equal after a long time. Neglecting the negative exponentials, that means

x 1(t) ≈ x 2(t)

or

x0 v0
2 ( ) (e )
e αt + e − αt ≈ αt − e − αt .

meichenlaub 2021-09-14 20:10:00


After a long time, the parts that go like e − αt will be very close to zero, so we can simply ignore them in analyzing what happens
after a long time.

meichenlaub 2021-09-14 20:10:08


What do you get for an answer to the question?

bfan0805 2021-09-14 20:11:03


v 0 = x 0α

AOPS81619 2021-09-14 20:11:03


v 0 = αx 0

Hiboy 2021-09-14 20:11:03


v_0 = x_0*alpha

s3v3n 2021-09-14 20:11:03


v 0 = αx 0

taniaban 2021-09-14 20:11:03


v 0 = αx 0

Zian2006 2021-09-14 20:11:03


v 0 = αx 0

DGA_Student_1 2021-09-14 20:11:03


sorry, I meant v 0 = x 0α

Dodgers66 2021-09-14 20:11:03


αx 0

mathtiger6 2021-09-14 20:11:03


v 0 = αx 0

mathisawesome01 2021-09-14 20:11:03


x0 v0
= , so v 0 = αx 0
2 2α

QMech 2021-09-14 20:11:03


v_0 = α * (x_0)

meichenlaub 2021-09-14 20:11:08


Dropping the decaying exponential terms, we have

meichenlaub 2021-09-14 20:11:10

x0 v0
e αt = e αt.
2 2α

meichenlaub 2021-09-14 20:11:12


We can cancel the exponential and solve for v 0, giving

v 0 = αx 0.

meichenlaub 2021-09-14 20:11:17


Does this answer check out dimensionally?

mathisawesome01 2021-09-14 20:11:55


Yes

bfan0805 2021-09-14 20:11:55


yes

mathisawesome01 2021-09-14 20:11:55


1
yes, α has units of
s

PhysKid11 2021-09-14 20:11:55


yes, alpha has units of 1/t

EthanTLee23 2021-09-14 20:11:55


Yes, because α has units of 1/seconds

s3v3n 2021-09-14 20:11:55


Yes, α has dimensions of T − 1

Zian2006 2021-09-14 20:11:55


Yes

Dodgers66 2021-09-14 20:11:55


yes, α has dimensions of 1 over time to make the exponent dimensionless

mathtiger6 2021-09-14 20:11:55


yes

QMech 2021-09-14 20:11:55


yes since alpha has units of inverse time

meichenlaub 2021-09-14 20:12:01


Yes, α has dimensions of T − 1, so it works.

meichenlaub 2021-09-14 20:12:11


The result we've seen here shows up fairly often.

meichenlaub 2021-09-14 20:12:14


Whenever an object is slightly displaced or pushed from a stable equilibrium point (a local minimum of potential energy), it
oscillates back and forth with its position as a function of time given by sin and cos functions.

meichenlaub 2021-09-14 20:12:20


When an object is slightly displaced or pushed from an unstable equilibrium point (a local maximum of potential energy), it
accelerates exponentially away from equilibrium with is position as a function of time given by sinh and cosh functions. This topic
is explored in greater detail in the oscillations lesson in next year's PhysicsWOOT class.

meichenlaub 2021-09-14 20:12:38


Consider the double Atwood machine shown below. Find the acceleration of the masses. The top pulley is fixed. The bottom pulley
is free to move up or down. The pulleys are massless and frictionless.

meichenlaub 2021-09-14 20:12:40


meichenlaub 2021-09-14 20:12:46
This problem will be quite a contrast from previous one. In the previous problem, it was very direct to obtain an equation of motion
- all we had to do with plug the given force into Newton's second law. Once we had the equation of motion, most of the work in the
problem was in solving the equation of motion and interpreting the solutions.

meichenlaub 2021-09-14 20:12:57


In this problem, what are we going to get instead of a single equation of motion?

s3v3n 2021-09-14 20:13:35


A system of equations

QMech 2021-09-14 20:13:35


a system of equations

Hiboy 2021-09-14 20:13:35


a system of equations

EL2024 2021-09-14 20:13:35


a system of equations?

mathisawesome01 2021-09-14 20:13:35


three equations

Hershey806 2021-09-14 20:13:35


equations for each mass

JohnOfMages 2021-09-14 20:13:35


a system of equations of motion

HappiHippo 2021-09-14 20:13:35


system of equations

meichenlaub 2021-09-14 20:13:39


There are three masses, so we can apply Newton's second law three times. In addition with any other equations we find, that
creates a system of equations instead of a single equation of motion. Our technical work will be in solving the system via algebra,
not in solving a single differential equation with calculus.

meichenlaub 2021-09-14 20:13:53


Let's begin with mass 1. If we apply Newton's second law, what do we get?

meichenlaub 2021-09-14 20:13:55


(You can use T 1 for the tension in the top string.)

AOPS81619 2021-09-14 20:14:50


ma = T 1 − mg

Chickenmom 2021-09-14 20:14:50


m1a=T1-m1g

QMech 2021-09-14 20:14:50


T_1 - mg = ma taking upward to be positive

HappiHippo 2021-09-14 20:14:50


T 1 − m 1g = ma 1 assuming that a 1 points upward

s3v3n 2021-09-14 20:14:50


T 1 − m 1g = m 1a 1

DGA_Student_1 2021-09-14 20:14:50


¨
T 1 − m 1g = m 1y 1

mathisawesome01 2021-09-14 20:14:50


T 1 − m 1g = m 1a 1 (assuming it accelerates upward)
bfan0805 2021-09-14 20:14:50
T 1 − m 1g = m 1a

EL2024 2021-09-14 20:14:50


T1=m1a-m1g

EthanTLee23 2021-09-14 20:14:50


T 1 − m 1g = m 1a

taniaban 2021-09-14 20:14:50


m 1a 1 = T 1 − mg

Dodgers66 2021-09-14 20:14:50


m 1a 1 = T 1 − m 1g

mathtiger6 2021-09-14 20:14:50


T 1 − m 1g = m 1a 1

meichenlaub 2021-09-14 20:14:54


Newton's second law, F = ma, is a law for the net force. There are two forces on mass 1: a gravitational force m 1g downward and a
tension force upward. Let's call the tension force T 1. Then the net force is T 1 − m 1g and Newton's second law says

T 1 − m 1g = m 1a 1.

meichenlaub 2021-09-14 20:15:05


Newton's second law for masses m 2 and m 3 works in the same way, so

meichenlaub 2021-09-14 20:15:07

T 1 − m 1g = m 1a 1
T 2 − m 2g = m 2a 2
T 2 − m 3g = m 3a 3

meichenlaub 2021-09-14 20:15:15


Is that enough equations to find the accelerations of the masses like the problem asks?

bfan0805 2021-09-14 20:15:44


no

EL2024 2021-09-14 20:15:44


no their are 6 variables and 3 equations

AOPS81619 2021-09-14 20:15:44


No

mathisawesome01 2021-09-14 20:15:44


Nooooo

PhysKid11 2021-09-14 20:15:44


no

No_u_ 2021-09-14 20:15:44


no

taniaban 2021-09-14 20:15:44


no

mathtiger6 2021-09-14 20:15:44


no, too many unknowns

Hiboy 2021-09-14 20:15:44


no cause there's 6 variables and 3 equations
Chickenmom 2021-09-14 20:15:44
no

meichenlaub 2021-09-14 20:15:55


No, it's not. That's three equations, but there are five unknown variables in them (a 1, a 2, a 3, T 1, T 2). So we need two more
equations.

meichenlaub 2021-09-14 20:16:07


Our first three equations all come from Newton's second law. What will be the source of our last two equations?

Pleaseletmewin 2021-09-14 20:16:33


conservation of string

AOPS81619 2021-09-14 20:16:33


conservation of string

mathisawesome01 2021-09-14 20:16:33


string conservation

DGA_Student_1 2021-09-14 20:16:33


the constraints

PhysKid11 2021-09-14 20:16:33


string length must be conserved

bfan0805 2021-09-14 20:16:33


conservation of string

taniaban 2021-09-14 20:16:33


conservation of string

Hershey806 2021-09-14 20:16:33


conservation of string

meichenlaub 2021-09-14 20:16:41


Our place to look is kinematics. Specifically, the assumption that the strings do not change their length.

meichenlaub 2021-09-14 20:16:53


People sometimes call this "conservation of string".

meichenlaub 2021-09-14 20:16:56


Let's begin with a simple Atwood machine.

meichenlaub 2021-09-14 20:16:59


meichenlaub 2021-09-14 20:17:02
How are the accelerations related here?

PhysKid11 2021-09-14 20:17:50


a_1=-a_2

EL2024 2021-09-14 20:17:50


a1=-a2

bfan0805 2021-09-14 20:17:50


a_1 = -a_2

Hershey806 2021-09-14 20:17:50


a1 = − a2

DGA_Student_1 2021-09-14 20:17:50


a1 = − a2

EthanTLee23 2021-09-14 20:17:50


They are equal in magnitude, opposite in direction

taniaban 2021-09-14 20:17:50


equal in magnitude

dagurvich 2021-09-14 20:17:50


a1=a2

Zian2006 2021-09-14 20:17:50


a1 = a2

Latnor 2021-09-14 20:17:50


The same

QMech 2021-09-14 20:17:50


Ẍ_1 = -Ẍ_2

HappiHippo 2021-09-14 20:17:50


a1 = − a2

s3v3n 2021-09-14 20:17:50


a1 = − a2

AOPS81619 2021-09-14 20:17:50


a1 = − a2

Hiboy 2021-09-14 20:17:50


a_1 = a_2

Zian2006 2021-09-14 20:17:50


a1 = − a2

mathisawesome01 2021-09-14 20:17:50


a1 = − a2

Dhillonr25 2021-09-14 20:17:54


their absolute values are the same but they have different signs

harinsrikanth 2021-09-14 20:17:54


They are equal magnitude

meichenlaub 2021-09-14 20:18:00


In this Atwood machine, every time mass m 1 rises 1 cm, mass m 2 falls 1 cm. So Δy 1 = − Δy 2. Taking two time derivatives,

a 1 = − a 2.
meichenlaub 2021-09-14 20:18:09
Looking back at our original problem, can we conclude

a2 = − a3

because masses m 2 and m 3 are also on opposite sides of a pulley?

Dodgers66 2021-09-14 20:19:15


no because teh pullye is accelerating

mathtiger6 2021-09-14 20:19:15


no, they are in an accelerating frame

Dodgers66 2021-09-14 20:19:15


no because the pulley is accelerating

DGA_Student_1 2021-09-14 20:19:15


No, because the pulley they are attached to can, itself, move

Zian2006 2021-09-14 20:19:15


No. The entire system of the two is accelerating

EthanTLee23 2021-09-14 20:19:15


No, because there is also the acceleration of the pulley

AOPS81619 2021-09-14 20:19:15


No, because the bottom pulley is also accelerating

Hiboy 2021-09-14 20:19:15


because they are both accelerating upwards

PhysKid11 2021-09-14 20:19:15


no because the pulley accelerates

meichenlaub 2021-09-14 20:19:28


No, we can't, because that pulley moves. And we can't apply the same type of kinematic equation we use in a simple Atwood's
machine to mass m 1, either, because although it's over a fixed pulley so the principle applies, there's no point on the other end of
the rope!

meichenlaub 2021-09-14 20:19:40


Let's fix that by simply adding one. I'll put it at the center of the bottom pulley (remember that pulley can move).

meichenlaub 2021-09-14 20:19:43


meichenlaub 2021-09-14 20:20:33
So what equation do we get involving a 1 now?

meichenlaub 2021-09-14 20:21:05


(You can use a P. )

bfan0805 2021-09-14 20:21:52


a1 = − aP

AOPS81619 2021-09-14 20:21:52


a1 = − aP

DGA_Student_1 2021-09-14 20:21:52


a1 = − aP

PhysKid11 2021-09-14 20:21:52


a_1=-a_P

EL2024 2021-09-14 20:21:52


a1=-ap

Dhillonr25 2021-09-14 20:21:52


-a1=Ap

JohnOfMages 2021-09-14 20:21:52


a1 = -aP

taniaban 2021-09-14 20:21:52


a1 = − aP

Zian2006 2021-09-14 20:21:52


a1 = − aP

Latnor 2021-09-14 20:21:52


A_1=-A_p

EthanTLee23 2021-09-14 20:21:52


a1 = − ap

meichenlaub 2021-09-14 20:21:56


It's

a 1 = − a p.

meichenlaub 2021-09-14 20:21:59


And do you see what equation relates a 2, a 3, and a P ?

mathisawesome01 2021-09-14 20:23:07


a2 + a3
aP =
2

Zian2006 2021-09-14 20:23:07


a 2 + a 3 = 2a P

taniaban 2021-09-14 20:23:07


a2 + a3
aP = 2

bfan0805 2021-09-14 20:23:07


(a_2 + a_3)/2 = a_P

Pleaseletmewin 2021-09-14 20:23:07


a2 + a3 = aP / 2
AOPS81619 2021-09-14 20:23:07
a 2 + a 3 = 2a P

Latnor 2021-09-14 20:23:07


a2 − ap = a3 + ap

meichenlaub 2021-09-14 20:23:23


We could imagine going into a frame that is itself accelerating at a P. In this frame, the pulley P is stationary. In fact, any object
′ ′
moving with acceleration a in an inertial frame moves with acceleration a ′ = a − a P in this frame. Also in this frame a 2 = − a 3 .
Transforming back to the original frame,

a 2 − a P = − (a 3 − a P).

meichenlaub 2021-09-14 20:23:47


So here are our equations:

meichenlaub 2021-09-14 20:23:48

T 1 − m 1g = m 1a 1
T 2 − m 2g = m 2a 2
T 2 − m 3g = m 3a 3
a1 = − aP
a 2 − a P = − (a 3 − a P)

meichenlaub 2021-09-14 20:24:17


We've added two equations. Do we have enough to solve the problem now?

taniaban 2021-09-14 20:24:46


no; extra variable

Hershey806 2021-09-14 20:24:46


no, we introduced a P

Latnor 2021-09-14 20:24:46


no, we also added a variable

AOPS81619 2021-09-14 20:24:46


No

DGA_Student_1 2021-09-14 20:24:46


No, because we've added another unknown, a P

mathisawesome01 2021-09-14 20:24:46


Need to find relationship between T 2 and T 1

meichenlaub 2021-09-14 20:24:51


Nope. We added two equations, but we also added one more unknown. Does anybody see what we can do to get one more
equation (without introducing another unknown)?

meichenlaub 2021-09-14 20:24:59


Hint: the lower pulley is massless, but we haven't used that information yet.

mathtiger6 2021-09-14 20:25:50


T 1 = 2T 2 because of force balance on lower pulley

QMech 2021-09-14 20:25:50


T_1 = 2T_2

Square_264 2021-09-14 20:25:50


T 1 = 2T 2
chyc 2021-09-14 20:25:50
Force balance on lower pulley

taniaban 2021-09-14 20:25:50


the net force on the lower pulley is 0 (f = ma on point P)

meichenlaub 2021-09-14 20:25:58


Let's go back to Newton's second law and apply it to the lower pulley.

meichenlaub 2021-09-14 20:26:06


The net force on this pulley must be zero because it is massless. There is a force T 1 upward on the pulley and a force 2T 2
downward on it, so

T 1 = 2T 2.

meichenlaub 2021-09-14 20:26:27


(If you aren't sure that the downward force is 2T 2, try using an argument based on Newton's third law.)

meichenlaub 2021-09-14 20:26:28


So our equations are

meichenlaub 2021-09-14 20:26:32

T 1 − m 1g = m 1a 1
T 2 − m 2g = m 2a 2
T 2 − m 3g = m 3a 3
a1 = − aP
a 2 − a P = − (a 3 − a P)
T 1 = 2T 2

meichenlaub 2021-09-14 20:26:36


Wow, six equations! But I think you can solve them. Go ahead and do it, then tell me what you get for a 1.

meichenlaub 2021-09-14 20:31:01


Okay, I don't think anyone quite got all the algebra sorted out, but that's not too big a deal.

meichenlaub 2021-09-14 20:31:06


It's a little long to write it all out, but we can repeatedly substitute one equation into the others to eliminate variables. Eventually,
we get

4m 2m 3 − m 1(m 2 + m 3)
a1 = g .
4m 2m 3 + m 1(m 2 + m 3)

meichenlaub 2021-09-14 20:31:15


4m 2m 3
That's a bit of a gnarly result. But suppose we make the substitution M = . Then it turns out our result can be written
m2 + m3

M − m1
a1 = g .
M + m1

meichenlaub 2021-09-14 20:31:25


What does that equation look like?

Hershey806 2021-09-14 20:31:51


acceleration of normal atwood machine

taniaban 2021-09-14 20:31:51


acceleration with a simple atwood machine
EL2024 2021-09-14 20:31:51
a 2 mass attwood machine?

QMech 2021-09-14 20:31:51


acceleration of a pulley with masses M & m_1

Latnor 2021-09-14 20:31:51


atwood machine acceleration

EL2024 2021-09-14 20:31:51


a single atwood machine

meichenlaub 2021-09-14 20:31:56


That's the equation for an ordinary Atwood machine with masses m 1 and M. In other words, the complex combination of a pulley
and masses m 2 and m 3 acts exactly like a single mass M as far as the motion of m 1 is concerned.

meichenlaub 2021-09-14 20:32:05


And what's M when m 2 = m 3 ?

HappiHippo 2021-09-14 20:32:41


2m 2 = 2m 3

mathisawesome01 2021-09-14 20:32:41


2m 2

Zian2006 2021-09-14 20:32:41


2m 2

EL2024 2021-09-14 20:32:41


2m_2 or 2m_3

bananaman321 2021-09-14 20:32:41


2m2

DGA_Student_1 2021-09-14 20:32:41


2m 2

PhysKid11 2021-09-14 20:32:41


2m_@

Hiboy 2021-09-14 20:32:41


2

theallpro1 2021-09-14 20:32:41


2m_2

EthanTLee23 2021-09-14 20:32:41


2m 2

PhysKid11 2021-09-14 20:32:41


2m_2

QMech 2021-09-14 20:32:41


2(m_2)

Nivek 2021-09-14 20:32:41


2m

taniaban 2021-09-14 20:32:41


2m 2

chyc 2021-09-14 20:32:41


2m 2

Latnor 2021-09-14 20:32:41


2m_2

meichenlaub 2021-09-14 20:32:45


4m 2m 2
According to the formula, M = = 2m 2. In this case, the masses m 2 and m 3 are balanced and don't slide back or forth over
m2 + m2
their pulley (assuming there was no initial sliding). Then, as intuition suggests, they act the same as a single mass with their
masses combined.

meichenlaub 2021-09-14 20:32:58


And what's M when m 2 = 0 ?

PhysKid11 2021-09-14 20:33:26


0

taniaban 2021-09-14 20:33:26


0

Chickenmom 2021-09-14 20:33:26


o

DGA_Student_1 2021-09-14 20:33:26


0

Latnor 2021-09-14 20:33:26


0

EL2024 2021-09-14 20:33:26


0

bananaman321 2021-09-14 20:33:26


0

mathisawesome01 2021-09-14 20:33:26


0

Hiboy 2021-09-14 20:33:26


0

A-Aban 2021-09-14 20:33:26


0

meichenlaub 2021-09-14 20:33:31


In that case the formula says M = 0. If m 2 = 0 it takes no force to accelerate m 2, which means there's no tension in the lower rope,
which means there's no downward force on the pulley P, which means mass m 1 doesn't feel any mass on the other side of the
Atwood machine (even if mass m 3 is not zero.)

meichenlaub 2021-09-14 20:38:50


The shell theorem in Newtonian gravity says that the gravitational field of a uniform sphere is identical to that of a point mass of
the same mass as the sphere, located at the center of the sphere. However, consider a point mass source P and a uniform sphere S
outside it. Is the force F P → S also the same as if S were a point mass of the same mass located at the center of S ?

meichenlaub 2021-09-14 20:39:00


Before you jump to a conclusion here, imagine a point mass next to a cube, and a dot at the center of the cube.

meichenlaub 2021-09-14 20:39:05


meichenlaub 2021-09-14 20:39:09
The force from the black point source onto the cube is not the same as the force from the black point source onto the red dot, even
if the red dot has the same mass as the cube.

meichenlaub 2021-09-14 20:39:21


So let's think about the shell theorem, as stated in the question,

meichenlaub 2021-09-14 20:39:23


"The gravitational field of a uniform sphere is identical to that of a point mass of the same mass as the sphere, located at the
center of the sphere."

meichenlaub 2021-09-14 20:39:31


In PhysicsPOOT (the fictional physics class of dubious merit), they make an interesting point about this. They say, sure, the
gravitational field from a uniform sphere is the same as that from a point. But that's different from saying that the gravitational
force on a sphere is the same as the force on a point. The first one is about the source - a spherical source and a point source are
equivalent. But the second one is about the "target" - the object that the force is on. So although a spherical source and a point
source are equivalent for calculating gravitational forces, a spherical target and a point target are not.

meichenlaub 2021-09-14 20:39:58


Is PhysicsPOOT correct? Is the force on a sphere different from the force on a point of the same mass located at the center of the
sphere? Please justify your answer!

EL2024 2021-09-14 20:42:06


the net force is the same by the shell theorem and newtons third law but the distribution of the force is not because of varying
distances in the sphere

taniaban 2021-09-14 20:42:06


no it's not because newton's third law says these two forces are equal in magnitude

chyc 2021-09-14 20:42:06


No due to Newton’s third law

meichenlaub 2021-09-14 20:42:11


No, the force on a sphere is not any different from the force on a point mass of the same mass as the sphere, located at the
sphere's center. They are the same force, and this is true no matter the source.

meichenlaub 2021-09-14 20:42:21


Imagine a sphere and a cube next to each other in space.

meichenlaub 2021-09-14 20:42:24

meichenlaub 2021-09-14 20:42:27


If we want to find the force on the cube, then by the shell theorem, it's the same as the force from a point. That's by the shell
theorem, since the gravitational field created by the sphere and the point are the same.

meichenlaub 2021-09-14 20:42:37


meichenlaub 2021-09-14 20:42:40
Now we can apply Newton's third law. In both cases, the force from the cube on the sphere/point must point opposite in direction
and with equal magnitude to the force from the sphere/point on the cube.

meichenlaub 2021-09-14 20:42:47

meichenlaub 2021-09-14 20:42:51


So not only are the forces from a sphere and a point the same, the forces on a sphere and a point must be the same as well, by
Newton's third law.

meichenlaub 2021-09-14 20:43:04


Next we'll think about an even-trickier situation.

meichenlaub 2021-09-14 20:43:07


Suppose an electron a is moving along the + x axis and electron b is moving along the + y axis. Both move at speed v. What is the
approximate direction of the magnetic force from electron a on electron b, and from electron b on electron a ?

meichenlaub 2021-09-14 20:43:16


Here's a picture:

meichenlaub 2021-09-14 20:43:20


meichenlaub 2021-09-14 20:43:24
Let's start with the electron moving along the x axis. We can think of a moving electron heuristically as being similar to a current.
Which direction is the current running?

dagurvich 2021-09-14 20:43:59


-x

DGA_Student_1 2021-09-14 20:43:59


− x direction

EL2024 2021-09-14 20:43:59


-x

AOPS81619 2021-09-14 20:43:59


In the negative x direction

QMech 2021-09-14 20:43:59


-x direction

Zian2006 2021-09-14 20:43:59


-x

HappiHippo 2021-09-14 20:43:59


negative x

Zian2006 2021-09-14 20:43:59


-x because electron is negative

mathtiger6 2021-09-14 20:43:59


negative x

meichenlaub 2021-09-14 20:44:03


Because an electron carries negative charge and a current is the direction of flow of positive charge, the current points to the left,
in the − x direction.

meichenlaub 2021-09-14 20:44:11


And using the right hand rule, what direction does the magnetic field of current associated with electron a point at the location of
electron b ?

mathtiger6 2021-09-14 20:44:48


into the page

HappiHippo 2021-09-14 20:44:48


into the page

Dhillonr25 2021-09-14 20:44:48


into the paper for y>0
Zian2006 2021-09-14 20:44:48
Into the page

Dodgers66 2021-09-14 20:44:48


-z

DGA_Student_1 2021-09-14 20:44:48


− z direction

meichenlaub 2021-09-14 20:44:54


If you point your right thumb along the − x axis and curl your fingers around, you'll see that on the + y axis, the fingers point into the
monitor, in the − z direction.

meichenlaub 2021-09-14 20:45:02

meichenlaub 2021-09-14 20:45:09


And what direction is the magnetic force on electron b ?

meichenlaub 2021-09-14 20:45:49


Whoa, don't forget that electrons have negative charge!

DGA_Student_1 2021-09-14 20:46:13


+ x direction

Zian2006 2021-09-14 20:46:13


Actually +x

HappiHippo 2021-09-14 20:46:13


positive x

QMech 2021-09-14 20:46:13


+x

EthanTLee23 2021-09-14 20:46:13


+x

meichenlaub 2021-09-14 20:46:18


By the right hand rule, it's to the right, in the + x direction.

meichenlaub 2021-09-14 20:46:23


meichenlaub 2021-09-14 20:46:29
And what direction is the magnetic force on electron a ?

HappiHippo 2021-09-14 20:47:16


by symmetry its gonna be positive y right

mathisawesome01 2021-09-14 20:47:16


upup

QMech 2021-09-14 20:47:16


+y

Hiboy 2021-09-14 20:47:16


+y

EthanTLee23 2021-09-14 20:47:16


+y

DGA_Student_1 2021-09-14 20:47:16


+ y direction

mathisawesome01 2021-09-14 20:47:16


+y

Dodgers66 2021-09-14 20:47:16


+y

mathtiger6 2021-09-14 20:47:16


+y

meichenlaub 2021-09-14 20:47:21


We can think of electron b as a current pointing in the − y direction. Then the magnetic field at electron a is in the + z direction. The
force on electron a is upward, in the + y direction.

meichenlaub 2021-09-14 20:47:25


meichenlaub 2021-09-14 20:47:35
So what's the problem here?

mathisawesome01 2021-09-14 20:48:35


newton's 3rd law...

Zian2006 2021-09-14 20:48:35


Conservation of momentum appears to be violated

Dodgers66 2021-09-14 20:48:35


newton's third law

DGA_Student_1 2021-09-14 20:48:35


the internal forces don't cancel --> the mechanical momentum of the system isn't conserved, even though it is isolated

mathisawesome01 2021-09-14 20:48:35


newton's 3rd law *looks like* it isn't satisfied

taniaban 2021-09-14 20:48:35


it seems like newton's third law doesn't apply

EL2024 2021-09-14 20:48:35


they don't add to 0?

AOPS81619 2021-09-14 20:48:35


They aren't in opposite directions

meichenlaub 2021-09-14 20:49:03


These forces don't obey Newton's third law!

meichenlaub 2021-09-14 20:49:08


(We can't make a firm conclusion about how they'll move because we only looked at magnetic forces, and we didn't calculate them
exactly. It's just a heuristic. But this is a real problem - the forces really aren't equal and opposite.)

meichenlaub 2021-09-14 20:49:16


Any guesses as to what's going on?

Zian2006 2021-09-14 20:50:12


The electromagnetic field has a momentum

AOPS81619 2021-09-14 20:50:12


Is it something with relativity?

DGA_Student_1 2021-09-14 20:50:12


momentum is stored in the E and B fields

Zian2006 2021-09-14 20:50:12


Radiation?

DGA_Student_1 2021-09-14 20:50:12


the "hidden" momentum is stored in the E and B fields

meichenlaub 2021-09-14 20:50:20


The magnetic forces on the electrons, although we have not calculated them exactly here, really do not sum to zero and don't obey
Newton's third law - it's not a trick. However, that doesn't mean that the basic idea behind Newton's third law is wrong, just that our
understanding is incomplete.

meichenlaub 2021-09-14 20:50:25


Let's think about it this way: Newton's third law can be thought of as a law about conservation of momentum. Since a force is a
flow of momentum, when forces are equal in magnitude and opposite in direction, it means there's as much momentum flowing
one way as another, and the net momentum is conserved.

meichenlaub 2021-09-14 20:50:41


In this example, it appears that there's net momentum going into the electrons due to magnetic forces. If we still believe in
momentum conservation, there must be additional momentum going somewhere.

meichenlaub 2021-09-14 20:50:44


The extra momentum is going into the electromagnetic field itself!

meichenlaub 2021-09-14 20:50:54


This example was just a little preview. Later on, when we study electromagnetism in detail, we'll return to the topic of momentum
being carried in electromagnetic fields.

meichenlaub 2021-09-14 20:51:06


An hourglass has a plug which allows you to prevent the sand from flowing. You place the hourglass on a scale, then remove the
plug so that sand flows. Sketch the reading on the scale as a function of time. You may assume the rate of flow of the sand is
roughly constant as it drains.

meichenlaub 2021-09-14 20:51:14


In physics problems, scales equilibrate infinitely quickly. A real scale could, for example, be made from a spring that bounces up
and down for a while before settling in to its equilibrium location, but we won't be worried about that.

meichenlaub 2021-09-14 20:51:49


What force does the reading on the scale show?

meichenlaub 2021-09-14 20:51:51


(Not the magnitude, just what is the interaction that a scale reads.)

PhysKid11 2021-09-14 20:52:33


normal force

EthanTLee23 2021-09-14 20:52:33


The normal force on the hourglass from the scale

DGA_Student_1 2021-09-14 20:52:33


the normal force on the scale by the hourglass

Dhillonr25 2021-09-14 20:52:33


the normal force

mathtiger6 2021-09-14 20:52:33


normal force exerted on hourglass

HappiHippo 2021-09-14 20:52:33


normal force

rohitk14 2021-09-14 20:52:33


normal force?

taniaban 2021-09-14 20:52:33


the normal force
meichenlaub 2021-09-14 20:52:42
The reading on the scale is the normal force between the hourglass and the scale. Before we let any of the sand start falling, this is
equal to the weight of the hourglass.

meichenlaub 2021-09-14 20:52:51


Once we open the hourglass, some sand starts falling. After sand starts falling but before any sand hits the bottom of the
hourglass, what happens to the reading on the scale? How do you know?

PhysKid11 2021-09-14 20:54:26


it decreases because the sand is in freefall and has 0 weight

Hiboy 2021-09-14 20:54:26


decreases

EL2024 2021-09-14 20:54:26


it's lower because part of the sand is in free fall so it cannot apply its weight

chyc 2021-09-14 20:54:26


reading goes down due to smaller normal force

taniaban 2021-09-14 20:54:26


the reading decreases because the hourglass is supporting less of the sand

Hiboy 2021-09-14 20:54:26


because the sand just lost contact with the hourgalss

harinsrikanth 2021-09-14 20:54:26


It decreases, as some sand must be in free fall

RAM12 2021-09-14 20:54:26


The reading decreases since the hourglass isn't holding as much of the sand

DGA_Student_1 2021-09-14 20:54:26


the reading on the scale abruptly decreases, as there must be a net downward force on the hourglass to account for the increase in
downward momentum of the sand

EthanTLee23 2021-09-14 20:54:26


The reading decreases

meichenlaub 2021-09-14 20:54:34


The reading goes down. One way to see this is that the sand that's falling in mid-air doesn't put any force on the hourglass, so the
hourglass doesn't feel its weight. But we can see this result in terms of the Newton's laws and the center of mass of the system as
well. Can you explain that point of view?

Zian2006 2021-09-14 20:56:50


The center of mass is accelerating downwards, so it felt less normal force

AOPS81619 2021-09-14 20:56:50


The center of mass is accelerating downwards, so the net force must be negative, so normal force must decrease.

DGA_Student_1 2021-09-14 20:56:50


the COM is accelerating downward, so there must be a net force downward. Hence the normal force on the hourglass by the scale
has to decrease to become less than the gravitational force downward on the sand + hourglass.

meichenlaub 2021-09-14 20:56:56


There are two forces on the hourglass+sand system: a gravitational force downward and a normal force upward. While the system
is sitting stationary with no sand falling, these forces are equal in magnitude and opposite in the direction. (But they are not a
Newton's third law pair.)

meichenlaub 2021-09-14 20:57:07


meichenlaub 2021-09-14 20:57:12
The net force on the hourglass+sand system is equal to its mass times the acceleration of its center of mass. While sand is falling
(and no sand has hit the bottom of the hourglass), the center of mass of the system is accelerating down. (It's accelerating at
much less than g because only a small part of the mass is accelerating down, but it's still accelerating down.) So during this period,
the net force on the hourglass+sand system is down.

meichenlaub 2021-09-14 20:57:26


The gravitational force on the system doesn't fluctuate, though. So in order for the net force to be down, the upward normal force
must decrease. This normal force is what the scale measures, so the reading on the scale goes down.

meichenlaub 2021-09-14 20:57:36


What happens to the scale reading when the sand starts hitting the bottom of the hourglass?

mathisawesome01 2021-09-14 20:58:39


scale reading abruptly jumps again

Pleaseletmewin 2021-09-14 20:58:39


there is an impulse

Zian2006 2021-09-14 20:58:39


Stablizes to a constant

Chickenmom 2021-09-14 20:58:39


It increases

rohitk14 2021-09-14 20:58:39


goes back up to original because now supporting same amount of mass?

Hiboy 2021-09-14 20:58:39


increases to normal

Dhillonr25 2021-09-14 20:58:39


does it increase because the sand imparts momentum into the hourglass which imparts it into the scale

HappiHippo 2021-09-14 20:58:39


the reading starts to increase

bfan0805 2021-09-14 20:58:39


it increases

Zian2006 2021-09-14 20:58:39


Becomes constant

meichenlaub 2021-09-14 20:58:44


The scale reading goes up when stand starts hitting the bottom. The sand hitting the bottom is accelerating upward, since it goes
from a negative velocity to zero velocity. This means there must be an upward force on the sand, and therefore a downward force
from the sand on the hourglass. The hourglass doesn't accelerate though, so the upward normal force from the scale must
increase, and the scale reading goes up.

meichenlaub 2021-09-14 20:59:10


We don't know yet whether the scale reading goes back exactly to the weight of the hourglass, or something a bit more, or
something a bit less. But can you give an argument that the scale reading is probably quite close to the weight of the hourglass
during this period based on Newton's second law?

Zian2006 2021-09-14 21:00:49


The sand is probably falling at close to a constant rate, so the center of mass is moving down at a constant velocity, and the net
force is 0

Dodgers66 2021-09-14 21:00:49


the situation is basically the same each momentum of time, just different amounts of sand in the top and bottom, so the momentum
is not changing much, meaning the net force is probably close to 0

meichenlaub 2021-09-14 21:00:52


The hourglass doesn't move. The sand falls, but during the draining process, the rate of falling sand is roughly constant, so the
average velocity of the sand isn't changing very much. That means there's very little acceleration of the system, and so by Newton's
second law the net force should be close to zero. That in turn implies that the reading on the scale is very close to the system's
weight.

meichenlaub 2021-09-14 21:01:06


There could be small fluctuations if, for example, the rate that sand flows through the hole slows down over time, or the amount of
sand falling at a given time decreases because the trip from the hole in the hourglass to the sand at the bottom is getting shorter.
But these variations can be expected to be pretty small.

meichenlaub 2021-09-14 21:01:40


Okay, what happens at the end, when the last of the sand has passed through the hole, but sand is still hitting the bottom of the
hourglass?

Zian2006 2021-09-14 21:02:23


Increases

AOPS81619 2021-09-14 21:02:23


The reading is more than the actual weight

HappiHippo 2021-09-14 21:02:23


the apparent weight increases a bit

Dodgers66 2021-09-14 21:02:23


normal force > gravity as center of mass has to come to stop

Zian2006 2021-09-14 21:02:23


Sharply increases

meichenlaub 2021-09-14 21:02:28


At that point, the reading on the scale goes up above the weight of the system because the average velocity of the sand is
increasing from some negative value to zero, so the sand is on average accelerating upward. That means the center of mass of the
system is accelerating upward, and a normal force from the scale is responsible.

meichenlaub 2021-09-14 21:02:45


So let's look at the reading on the scale over time.

meichenlaub 2021-09-14 21:02:47


It has a little negative hump when the sand starts falling, then a positive hump at the end. At other times, it's close to the weight of
the hourglass.

meichenlaub 2021-09-14 21:02:50


meichenlaub 2021-09-14 21:02:58
Tell me about the time-average reading on the scale. What is it and why?

Zian2006 2021-09-14 21:04:51


mg because in the end the entire system is stationary

EL2024 2021-09-14 21:04:51


the average reading should be the same as it's weight because the velocity of the center of mass after is 0

DGA_Student_1 2021-09-14 21:04:51


time average reading = area under curve divided by time interval = mg

taniaban 2021-09-14 21:04:51


the weight of the hourglass + sand because the net impulse is 0

Dodgers66 2021-09-14 21:04:51


just the weight as there is no change in momentum from the start and end

EthanTLee23 2021-09-14 21:04:51


Its equal to the normal weight of the hourglass, because the positive and negative bumps cancel out.

meichenlaub 2021-09-14 21:04:55


The time-average reading is simply the weight of the hourglass+sand system. The reason is simple: the system starts with zero
momentum and ends with zero momentum. That means the change in its momentum is zero. Since the change in the momentum
of a system is the time average of the net force on the system,

Δp = ∫ F netdt,

the time-average net force must be zero. The time-average normal force from the scale therefore has to be exactly equal to the
weight of the system.

meichenlaub 2021-09-14 21:05:20


A rocket sits stationary in deep space. The rocket has a fraction α of its mass in fuel. The rocket can eject fuel at a speed v 0
relative to the rocket. What is the maximum speed of the rocket?

meichenlaub 2021-09-14 21:05:29


In this problem, we will derive the famous Tsiolkovsky rocket equation.

meichenlaub 2021-09-14 21:05:34


So the rocket begins, sitting still in deep space, and starts shooting mass out from its backside at speed v 0. What further
information will you need to figure out the acceleration of the rocket?

taniaban 2021-09-14 21:06:25


the mass flow rate
Zian2006 2021-09-14 21:06:25
The rate of ejection of mass

EL2024 2021-09-14 21:06:25


mass of rocket, rate of mass ejection

QMech 2021-09-14 21:06:25


mass of the rocket and ejected mass to determine the change in momentum

Hiboy 2021-09-14 21:06:25


the mass of the shooting-out stuff

Chickenmom 2021-09-14 21:06:25


The change of the mass of the rocket

meichenlaub 2021-09-14 21:06:59


You'll need to know how much mass per unit time is being fired out. If the mass of the rocket is M(t) we can call the rate that fuel is
dM
fired out − .
dt

meichenlaub 2021-09-14 21:07:15


What's the force on the rocket?

Dhillonr25 2021-09-14 21:08:06


dM/dt*v0

EL2024 2021-09-14 21:08:06


dM/dt*v0

Hershey806 2021-09-14 21:08:06


dM
v0
dt

PhysKid11 2021-09-14 21:08:06


vdM/dt

Dodgers66 2021-09-14 21:08:06


dM
v 0 dt

HappiHippo 2021-09-14 21:08:06


dM
v⋅
dt

taniaban 2021-09-14 21:08:06


dM
v 0 dt

QMech 2021-09-14 21:08:06


(v_0)dM/dt

Zian2006 2021-09-14 21:08:06


dM
− dt ∗ v 0

meichenlaub 2021-09-14 21:08:14


The rocket is putting momentum per unit time

( )
dp fuel dM
= ( − v 0) ⋅ −
dt dt

into the fuel. (Here we're setting the velocity of the fuel to be − v 0 so that the rocket's velocity is positive. There is also a negative
dM
sign on the term because the negative of the rate of change of mass of the rocket is the mass of fuel per second ejected.) This
dt
is equal to the force on the fuel because a force is the flux of momentum into a system.
meichenlaub 2021-09-14 21:08:23
By Newton's third law, the fuel is putting an equal and opposite force back on the rocket, so

dM
F = − v0 .
dt

meichenlaub 2021-09-14 21:08:31


Using Newton's second law, we can rewrite that as

dv dM
M = − v0 .
dt dt

meichenlaub 2021-09-14 21:08:44


Earlier, we said that to find the acceleration, you need to know the mass per unit time that the rocket is firing. Looking at this most-
recent equation, does the final velocity of the rocket depend on the rate at which it fires mass?

Dodgers66 2021-09-14 21:09:28


no

mathtiger6 2021-09-14 21:09:28


no

EL2024 2021-09-14 21:09:28


no

Zian2006 2021-09-14 21:09:28


No, since the dt's cancel

QMech 2021-09-14 21:09:28


No since dt cancels out

meichenlaub 2021-09-14 21:09:32


dM
No. The final velocity doesn't depend on how fast we fire mass. If we fire mass twice as fast, for example, we double . This
dt
doubles dv / dt as well. Everything is the same as before, but it's as if we've introduced a new time coordinate t ′ = t / 2. In other
words, we'll get to the final velocity of the rocket twice as fast, but the final velocity will be unchanged.

meichenlaub 2021-09-14 21:09:48


In the equation, we can essentially "cancel the dt ′ s" to get

Mdv = − v 0dM.

meichenlaub 2021-09-14 21:10:01


Why does this work? Suppose we think of M and v as functions of t. Then we could write v as

v(t) = v(M(t)).

Using the chain rule,

dv dv dM
= .
dt dM dt

dv dM
Plugging this in to M = − v0 we get
dt dt

dv dM dM
M = − v0 .
dM dt dt

meichenlaub 2021-09-14 21:10:16


That simplifies to
Mdv = − v 0dM

as we had before.

meichenlaub 2021-09-14 21:10:25


So what do you get for v(M) ?

mathisawesome01 2021-09-14 21:12:02

v(M) = v 0ln
M0
M ( )
Dodgers66 2021-09-14 21:12:02
v 0ln(M 0 / M)

QMech 2021-09-14 21:12:02


v(M) = (v_0)In((M_0)/M) where M_0 is the initial mass of the rocket

meichenlaub 2021-09-14 21:12:07


Separating variables,

dM
dv = − v 0 .
M

meichenlaub 2021-09-14 21:12:10


Integrating,

v = − v 0ln
( )
M
M0
.

Here, we've chosen the constant M 0 so that v(M 0) = 0.

meichenlaub 2021-09-14 21:12:24


And what's the maximum speed of the rocket?

mathisawesome01 2021-09-14 21:13:49


1
at the end, M = (1 − α)M 0, so v max = v 0ln
1−α

Zian2006 2021-09-14 21:13:49


1
v 0ln( 1 − α )

DGA_Student_1 2021-09-14 21:13:49


v max = − v 0ln(1 − α)

taniaban 2021-09-14 21:13:49


− v 0ln(1 − α)

mathtiger6 2021-09-14 21:13:49


− v 0ln(1 − α)

Dodgers66 2021-09-14 21:13:49


− v 0ln(1 − α)

meichenlaub 2021-09-14 21:13:54


It's

v max = − v 0ln(1 − α).


meichenlaub 2021-09-14 21:13:59
This might be more insightful if we solve if for α.

meichenlaub 2021-09-14 21:14:05


That's

α = 1 − e − v / v 0.

meichenlaub 2021-09-14 21:14:09


In theory, there's no limit to your maximum speed. You could always make α closer and closer to one and get a higher and higher
maximum speed. But in practice, if you want to go just a few times faster than the speed at which you expel propellant, your rocket
needs to be essentially all fuel. Can you give an intuitive reason why, once your rocket has a lot of fuel, adding more and more fuel
doesn't really increase the final speed much?

HappiHippo 2021-09-14 21:15:09


the rocket needs to carry that fuel

EL2024 2021-09-14 21:15:09


adding more fuel increases weight

Hiboy 2021-09-14 21:15:09


because you'll need to propell for fuel's weight

HappiHippo 2021-09-14 21:15:09


its more fuel for the rocket to lift off

PhysKid11 2021-09-14 21:15:09


because the mass of the rocket itself increases so it's harder to accelerate

Zian2006 2021-09-14 21:15:09


You need more fuel to push itself

meichenlaub 2021-09-14 21:15:15


Suppose your rocket is 99% fuel already. You double the amount of fuel. Now you have twice as much fuel, but your rocket is very
nearly twice as heavy! So you have to burn a lot more fuel just to get up to the same velocity as before. The extra fuel will pay off in
the end in terms of increasing your final speed, but not by much because you have to spend so much energy just accelerating all
the extra fuel you took on.

meichenlaub 2021-09-14 21:15:32


In practice, if you want to make a rocket go very fast, you'll probably be better off finding ways to increase v 0 instead of increasing
α.

meichenlaub 2021-09-14 21:15:40


Accelerating reference frames

meichenlaub 2021-09-14 21:15:44


You're sitting in a stationary train holding a helium balloon by the string. The train starts accelerating forward. Which direction
does the balloon tilt?

meichenlaub 2021-09-14 21:15:46


meichenlaub 2021-09-14 21:15:51
Let's start by analyzing the balloon with Newton's laws. Here's a free body diagram for it before the acceleration. The net force on
the balloon is zero.

meichenlaub 2021-09-14 21:15:57

meichenlaub 2021-09-14 21:16:03


Once the train starts accelerating forward, what direction is the net force on the balloon?

mathisawesome01 2021-09-14 21:16:43


goes to the right

PhysKid11 2021-09-14 21:16:43


rightward

EthanTLee23 2021-09-14 21:16:43


Forwards

Nivek 2021-09-14 21:16:43


forward

rohitk14 2021-09-14 21:16:43


rightwards?

mathisawesome01 2021-09-14 21:16:43


right

DGA_Student_1 2021-09-14 21:16:43


forward, in the same direction as the acceleration

aj1745 2021-09-14 21:16:43


forward

meichenlaub 2021-09-14 21:16:49


The balloon is in the train, so it's also accelerating forward. The net force on the balloon is forward.

meichenlaub 2021-09-14 21:16:59


Where does that net force come from?

Zian2006 2021-09-14 21:17:25


$T4

EL2024 2021-09-14 21:17:25


tension
Zian2006 2021-09-14 21:17:25
T

EthanTLee23 2021-09-14 21:17:25


Tension in the string

taniaban 2021-09-14 21:17:25


the air

Dodgers66 2021-09-14 21:17:25


air pressure

Dodgers66 2021-09-14 21:17:25


and string

AOPS81619 2021-09-14 21:17:25


The air

meichenlaub 2021-09-14 21:17:31


There seem to be two camps here. One group says that forward force on the balloon comes from the tension in the string. The
other says that it comes from the air.

meichenlaub 2021-09-14 21:17:40


Suppose the group saying it's tension in the string that pulls the balloon forward is right. For that to be the case, which way would
the balloon tilt - towards the back of the train or towards the front of it?

Chickenmom 2021-09-14 21:18:09


towards the back

EL2024 2021-09-14 21:18:09


toward the back

HappiHippo 2021-09-14 21:18:09


towards the back

EthanTLee23 2021-09-14 21:18:09


Back

DGA_Student_1 2021-09-14 21:18:09


towards the back

meichenlaub 2021-09-14 21:18:14


If tension is pulling the balloon forward, then we can see from the following diagram that the balloon is tilting backwards, toward
the back of the train.

meichenlaub 2021-09-14 21:18:17


meichenlaub 2021-09-14 21:18:25
And if air is pushing the balloon forward, then the forward force from the air will make the balloon tip forward, toward the front of
the train.

meichenlaub 2021-09-14 21:18:34


Let's try to resolve the uncertainty by using what we learned last week and treating this as a statics problem. What do we have to
do to turn this dynamics problem (a problem about an accelerating balloon) into a statics problem (a problem about a stationary
balloon)?

QMech 2021-09-14 21:19:27


Switch to the frame of the train

PhysKid11 2021-09-14 21:19:27


change frames of reference

mathisawesome01 2021-09-14 21:19:27


move into the accelerating frame

rohitk14 2021-09-14 21:19:27


change reference frame

Zian2006 2021-09-14 21:19:27


Moving into accelerating frame by introducing psuedoforce

EthanTLee23 2021-09-14 21:19:27


Use the reference frame of the balloon

A-Aban 2021-09-14 21:19:27


change reference frame to balloon?

DGA_Student_1 2021-09-14 21:19:27


change into the accelerating frame and add a fictitious force − ma

taniaban 2021-09-14 21:19:27


use an accelerating reference frame

Dodgers66 2021-09-14 21:19:27


accelerating reference frame

AOPS81619 2021-09-14 21:19:27


Switch to the frame of the train

rohitk14 2021-09-14 21:19:27


prob reference frame of the train

EL2024 2021-09-14 21:19:27


switch frames?
HappiHippo 2021-09-14 21:19:27
add a fictitious force

Pleaseletmewin 2021-09-14 21:19:27


F fict = − ma train

Hiboy 2021-09-14 21:19:27


adding the acceleration of train as a backward force on balloon

meichenlaub 2021-09-14 21:19:33


We need to go into an accelerating reference frame. Our frame should be accelerating forward along with the train.

meichenlaub 2021-09-14 21:19:38


In the frame accelerating forward at acceleration a, an object of mass m experiences an inertial force backward with magnitude
ma.

meichenlaub 2021-09-14 21:19:45


So the balloon experiences a backward inertial force and tilts backward, with the string pulling it forward, right?

meichenlaub 2021-09-14 21:19:47

taniaban 2021-09-14 21:20:56


no because the air is accelerating as well

EL2024 2021-09-14 21:20:56


isn't buoyancy also restricted to the opposite direction of gravity?

mathisawesome01 2021-09-14 21:20:56


nooo buoyancy

meichenlaub 2021-09-14 21:21:01


Not so fast! The balloon does experience a backward inertial force, but it also experiences a force from the air which we haven't
analyzed yet.

meichenlaub 2021-09-14 21:21:05


What other principle did we learn about inertial forces that might make things conceptually clearer here?

DGA_Student_1 2021-09-14 21:21:53


we can treat them as gravitational forces

Dodgers66 2021-09-14 21:21:53


effective gravity

meichenlaub 2021-09-14 21:21:57


An inertial force is indistinguishable from a uniform gravitational field. So in the accelerating frame, what's the effective direction
of gravity?

mathisawesome01 2021-09-14 21:23:01


left and down

Zian2006 2021-09-14 21:23:01


Down and to the left

DGA_Student_1 2021-09-14 21:23:01


down and left

QMech 2021-09-14 21:23:01


down and to the left

Hershey806 2021-09-14 21:23:01


down and back

Dhillonr25 2021-09-14 21:23:01


slightly to the left (from the vertical)

HappiHippo 2021-09-14 21:23:01


down and slightly to the left

Hiboy 2021-09-14 21:23:01


bottom left corner?

theallpro1 2021-09-14 21:23:01


arctan(a/g) from the vertical

taniaban 2021-09-14 21:23:01


down and to the left

meichenlaub 2021-09-14 21:23:07


The actual gravitational force points downward, and the inertial force points backward, so the effective gravity points down and
backwards.

meichenlaub 2021-09-14 21:23:12

meichenlaub 2021-09-14 21:23:17


And what does a balloon do, relative to gravity? Therefore, which direction does the balloon tilt?

EthanTLee23 2021-09-14 21:24:12


Forward

taniaban 2021-09-14 21:24:12


goes opposite, so it tilts forward

HappiHippo 2021-09-14 21:24:12


it goes in the opposite direction

Hershey806 2021-09-14 21:24:12


opposite

mathisawesome01 2021-09-14 21:24:12


to the right

DGA_Student_1 2021-09-14 21:24:12


it tilts opposite the effective gravitational field, i.e., to the right at some angle

mathtiger6 2021-09-14 21:24:12


tilts rightward, opposite to effective gravity

Square_264 2021-09-14 21:24:12


tilt forwards

Zian2006 2021-09-14 21:24:12


Actually, forwards

Dodgers66 2021-09-14 21:24:12


string aligns itself with gravity

meichenlaub 2021-09-14 21:24:16


A balloon points in the opposite direction as gravity! So the balloon tilts forward towards the front of the train.

meichenlaub 2021-09-14 21:24:22


But this seems kind of strange. In the accelerating frame, there's an inertial force backward on the balloon, yet the balloon tilts
forward. What could be providing the force pushing the balloon forward?

Chickenmom 2021-09-14 21:25:14


air pressure

aj1745 2021-09-14 21:25:14


air

Dhillonr25 2021-09-14 21:25:14


bouyancy

QMech 2021-09-14 21:25:14


Buoyant force from the air

DGA_Student_1 2021-09-14 21:25:14


the air density on the left side of the compartment is higher than on the right side. this creates a pressure gradient, with the higher
pressure on the left, such that the balloon experiences a net push right

Hiboy 2021-09-14 21:25:14


air pressure from the air that concentrated at the bottom left corner

AOPS81619 2021-09-14 21:25:14


buoyant force

mathisawesome01 2021-09-14 21:25:14


air pressure

HappiHippo 2021-09-14 21:25:14


is it bouyancy?

taniaban 2021-09-14 21:25:14


the air wants to go backward so it pushes the balloon forward

meichenlaub 2021-09-14 21:25:19


It must be the air, since the string pulls backward along with the inertial force.
meichenlaub 2021-09-14 21:25:25

meichenlaub 2021-09-14 21:25:34


In the accelerating frame, there is an inertial force pointing backwards on any little patch of air. The air is not accelerating in this
frame, though, so the net force on any little patch of air is zero. This means there is a forward force of magnitude ma on any patch
of air of mass m. In terms of pressure, the pressure increases toward the back of the train. This creates a pressure gradient which
exerts a force on the air.

meichenlaub 2021-09-14 21:26:10


In fact, the transient effect here is quite noticeable. By the transient effect, I mean what happens between the first equilibrium,
when the train is stationary, and the second equilibrium, when the train has constant acceleration and everything is settled down.

meichenlaub 2021-09-14 21:26:12


Right when the train starts accelerating, there is a backward inertial force on the air, but the air hasn't developed a pressure
gradient yet. This means the air starts accelerating backward toward the back of the train until there's enough air further back to
create a pressure gradient that cancels the inertial force. You could experience this as a wind blowing towards the back of the
train.

meichenlaub 2021-09-14 21:26:35


Back to the equilibrium case with the accelerating train. There is a pressure gradient in the air. This causes a force ma forward on
any patch of air of mass m. It causes the same force on a balloon, where m is now that mass of air that the balloon displaces. (I.E.
the mass that the air would have if you filled the balloon with air.) Because the balloon is lighter than air, the force on it from the
pressure gradient in the air is greater than the inertial force pushing it backward, and the balloon goes forward.

meichenlaub 2021-09-14 21:27:04


Okay, we'll do one last problem.

meichenlaub 2021-09-14 21:27:08


A baseball pitcher has spent their entire career pitching at latitudes of around 45 ∘ N. The pitcher decides to play an exhibition
match in Argentina, at a latitude of 45 ∘ S. A typical baseball pitch has a speed of about 40 m / s and travels about 18 m from the
pitcher to the batter.

meichenlaub 2021-09-14 21:27:15


Is the drift large enough that the pitcher needs to worry about it?

meichenlaub 2021-09-14 21:27:24


In case you aren't familiar with baseball, here's the phenomenon being described:

meichenlaub 2021-09-14 21:27:26


meichenlaub 2021-09-14 21:27:30
The man throwing the ball is the pitcher and the man catching it is the catcher. The man holding the stick (called a "bat") is the
batter. The gif shows a right-handed batter.

meichenlaub 2021-09-14 21:27:38


"Outside" means "away from the batter" and inside means "towards the batter". The pitch shown is very much inside.

meichenlaub 2021-09-14 21:27:45


Why might the pitcher's pitch be affected by playing in the southern hemisphere?

rohitk14 2021-09-14 21:28:02


are you referring to Coriolis acceleration?

HappiHippo 2021-09-14 21:28:02


coriolis effect?

Dodgers66 2021-09-14 21:28:02


coriolis

Hershey806 2021-09-14 21:28:02


coriolis

DGA_Student_1 2021-09-14 21:28:02


coriolis force

Dhillonr25 2021-09-14 21:28:02


Coriolis force?

EL2024 2021-09-14 21:28:02


coriolis effect

meichenlaub 2021-09-14 21:28:06


In the southern hemisphere, the Coriolis force points in the opposite direction compared to the northern hemisphere.

meichenlaub 2021-09-14 21:28:11


In the reading, we saw that the Coriolis force is described by the equation

→ → →
F C = − 2mω × v .

meichenlaub 2021-09-14 21:28:16


In this problem, ω is the angular velocity vector of Earth. We'll talk more about this vector next week, but for now, we need to know
that it points out of the north pole.

meichenlaub 2021-09-14 21:28:26


What's its magnitude?

AOPS81619 2021-09-14 21:29:01



86400

Dhillonr25 2021-09-14 21:29:01


the angular velocity of earth, which is about pi/12ish rad per hour

EL2024 2021-09-14 21:29:01


2pi/(3600*24)

Hershey806 2021-09-14 21:29:01



length of day

meichenlaub 2021-09-14 21:29:06


The Earth rotates once in about 24 h, so

→ 2π
|ω| ≈ ≈ 7.3 × 10 − 5s − 1.
24 h

meichenlaub 2021-09-14 21:29:17


What do you get for the acceleration of the baseball due to the Coriolis force, assuming it's traveling north-south? (This is the
direction where the effect is most significant.)

mathtiger6 2021-09-14 21:30:23


0.004 m/s^2

meichenlaub 2021-09-14 21:30:29


It's

F
a= = 2ωvsinθ ≈ 4 × 10 − 3 m / s.
m

meichenlaub 2021-09-14 21:30:37


Did you forget about the sinθ part of a cross product? Did you never even hear about it before?

meichenlaub 2021-09-14 21:30:39


That's okay, but go ahead and learn / review what cross products are. We'll use them again next week for rotation, and will use
them more in electromagnetism. There a basic reference in last week's handout, or you could google for articles or videos that
teach it. There's one by "Three Blue One Brown" on YouTube that does a good job of showing you what it's about.

meichenlaub 2021-09-14 21:30:51


So does the pitcher need to worry about the Coriolis effect?

mathisawesome01 2021-09-14 21:31:30


no

mathtiger6 2021-09-14 21:31:30


no

QMech 2021-09-14 21:31:30


No

Chickenmom 2021-09-14 21:31:30


no

Zian2006 2021-09-14 21:31:30


Probably not
Dhillonr25 2021-09-14 21:31:30
probably not

AOPS81619 2021-09-14 21:31:30


no, the difference is less than a centimeter

taniaban 2021-09-14 21:31:30


no

HappiHippo 2021-09-14 21:31:30


no

Dhillonr25 2021-09-14 21:31:30


unless his pitch is so precise that a few millimeters will change it

meichenlaub 2021-09-14 21:31:34


The pitch takes about half a second, so the Coriolis deflection is about

1 1
Δx ≈ a(Δt) 2 ≈ (4 × 10 − 3 m / s) ⋅ (0.5 s) 2 ≈ 5 × 10 − 4 m = 0.5 mm.
2 2

meichenlaub 2021-09-14 21:31:39


Baseball pitchers are very accurate, but none are so accurate that a millimeter really matters, so no, the pitcher doesn't have to
worry about the Coriolis effect.

meichenlaub 2021-09-14 21:31:49


This result makes analyzing the Coriolis deflection further a moot point from a practical point of view, but we can still learn a little
something from answering a couple more questions.

meichenlaub 2021-09-14 21:31:52


Which type of pitch experiences a greater drift (in centimeters) - a slow one or a fast one?

Nivek 2021-09-14 21:32:21


slo

Dhillonr25 2021-09-14 21:32:21


slow

Nivek 2021-09-14 21:32:21


slow

AOPS81619 2021-09-14 21:32:21


slow

A-Aban 2021-09-14 21:32:21


slow

taniaban 2021-09-14 21:32:21


slow

Zian2006 2021-09-14 21:32:21


Slow one

PhysKid11 2021-09-14 21:32:21


a slow one since t is squared

meichenlaub 2021-09-14 21:32:27


Suppose the pitcher doubles their pitching speed. Then from the formula a = 2ωvsinθ, the Coriolis acceleration of the ball doubles.
The time over which that acceleration acts is cut in half, though. The sideways velocity imparted by the Coriolis force is
proportional to aΔt, and since a doubles and Δt is cut in half, the ball picks up about the same horizontal velocity during the pitch
regardless of its speed.

meichenlaub 2021-09-14 21:32:35


That means the fast pitch is deflected less. It picks up the same horizontal velocity as a slow pitch, but has less time to be pushed
by that horizontal velocity, and so is pushed less far. Doubling the pitch speed cuts the Coriolis deflection in half.

meichenlaub 2021-09-14 21:32:50


When in the southern hemisphere, pitching in the north-south direction, will the pitcher's pitches tend to drift more to the "outside"
or "inside" vs a right-handed batter, compared to when in the northern hemisphere?

mathtiger6 2021-09-14 21:34:04


outside

mathisawesome01 2021-09-14 21:34:04


outside

QMech 2021-09-14 21:34:04


outside

HappiHippo 2021-09-14 21:34:04


outside

meichenlaub 2021-09-14 21:34:08


Applying the right-hand rule, in the northern hemisphere, a pitch to the north is deflected east. This is "inside" for a right-handed
batter. In the southern hemisphere, it is reversed.

meichenlaub 2021-09-14 21:34:14


Summary

meichenlaub 2021-09-14 21:34:16


Newton's laws form the core of mechanics; many problems involve applying all three laws.

meichenlaub 2021-09-14 21:34:19



Newton's second law, F net = m→
a , often does the heavy lifting in mathematical problem solving because the equation F = ma
becomes an equation of motion.

meichenlaub 2021-09-14 21:34:22


Newton's laws can be applied to the center of mass of a complex system.

meichenlaub 2021-09-14 21:34:24

→ → → d→
p
In addition to the form F net = m a , another useful form of Newton's second law is F = . One can usually use either law, but
dt
practicing with both may make it easier to choose one that simplifies your work in a given problem.

meichenlaub 2021-09-14 21:34:28


In accelerating frames, Newton's laws are modified by introducing inertial forces. In uniformly-accelerating frames, this is a force
of magnitude ma pointing opposite the direction the frame is accelerating relative to an inertial frame. In a rotating frame, both a
centrifugal and Coriolis force are applied.

meichenlaub 2021-09-14 21:34:36


Credits:
Pulley Problem: Morin, Problems and Solutions in Introductory Mechanics, 2014.
Conservation of string length: http://www.people.fas.harvard.edu/~djmorin/ProblemsChap4.pdf

© 2022 Art of Problem Solving


About Us • Contact Us • Terms • Privacy

Copyright © 2022 Art of Problem Solving

You might also like